Witch is greater 100in or 3yd 1ft

Answers

Answer 1
Answer: First you need to convert then to the same units. So 3yd 1ft would be equal to 10 ft. Then you need to convert the feet to inches. 10ft x 12 is equal to 120 inches.

120 inches is greater that 100 inches.

Related Questions

How can I remember slope?
Find zHelp me please
Solve 3x+1(not on a graph do it using algebra) no jokes you will be deleted
I need to know. is this a function? yes or no? y = 1/4x
The diagram shows a patio in the shape of a rectangle.The patio is 3.6m long and 3m wide. Mathew is going to cover the patio with paving slabs. Each paving slab is a square of side 60cm. Mathew buys 32 of the paving slabs. A) Does Mathew buy enough paving slabs to cover the patio? The paving slabs cost £8.63 each B) Work out the total cost of the 32 paving slabs.

A box plot was made to represent the number of matches won by 14 participants in a tennis tournament. The box plot had the box shifted to the right so that the left tail was much longer than the right tail. Based on the box plot, which conclusion is correct? (A)The mean and median of matches won are equal.
(B)The mean of matches won is more than the median of matches won.
(C)Most of the participants won many matches, but there were some participants who won very few matches compared to the others.
(D)Most of the participants won very few matches, but there were some participants who won many matches compared to the others.

Answers

Answer:

(C)- Most of the participants won many matches, but there were some participants who won very few matches compared to the others.

Step-by-step explanation:

The Box plot isshifted to the right this means it is right-skewed data thus left tail is much longer than right tail. Hence, Option (C) is correct.

Option (A) means than neither right tail is longer nor left, the box will remain in middle only. So it is incorrect.

Option (B) is incorrect because Box Plot doesn't give us idea about mean.

Option (D) is also not correct because if this happened then box will shifted to the left

The box plot had the box shifted to the right so that the left tail was much longer than the right tail. Based on the box plot, the conclusion that is correct is Most of the participants won very few matches, but there were some participants who won many matches compared to the others. The answer is letter D

the sum of three consecutive integer is 88. identify the variable and write an equation the describes the situation.

Answers

Lets say that you take n as the integer. Now you'll add it with 3 numbers. n+(n+1)+(n+2)=88. Now add 'em up, the sum is 3n+3=88. Take the 3 to the other side and subtract it from 88 which is 85. Now it'll be n= 85/3

dan painted 3/4 of a wall using 1/4 of a can of paint. how many walls can he paint using one can of paint?

Answers

Using 1 can of paint 3 walls can be painted.

What is Unitary Method?

The unitary technique involves first determining the value of a single unit, followed by the value of the necessary number of units.

For example, Let's say Ram spends 36 Rs. for a dozen (12) bananas.

12 bananas will set you back 36 Rs. 1 banana costs 36 x 12 = 3 Rupees.

As a result, one banana costs three rupees. Let's say we need to calculate the price of 15 bananas.

This may be done as follows: 15 bananas cost 3 rupees each; 15 units cost 45 rupees.

Dan painted 3/4 of a wall using 1/4 of a can of paint.

So, 1/4 can used = 3/4 of wall

Then, in one can the amount of wall painted

= 3/4 x 4/1

= 3 walls.

Learn more about Unitary Method here:

brainly.com/question/22056199

#SPJ7

3/4         x
 ÷     =   ÷
1/4        1

(4)  1/4x = 3/4  (4)

1x=3

final answer: dan can paint 3 walls

The label on the car's antifreeze container claims to protect the car between −40°C and 125°C. To convert Celsius temperature to Fahrenheit temperature, the formula is C = 5 over 9 (F − 32). Write and solve the inequality to determine the Fahrenheit temperature range at which this antifreeze protects the car.−40 > 5 over 9 (F − 32) > 125; −40 > F > 257

−40 < 5 over 9 (F − 32) < 125; −40 < F < 257

−40 < 5 over 9 (F − 32); −40 < F

5 over 9 (F − 32) < 125; F < 257

Answers

Given
Between -40°C and 125°C

°C = 5/9 * (°F-32)
°C ÷ 5/9 = °F - 32
°C * 9/5 + 32 = °F

°F = -40°C * 9/5 + 32 = -360/5 + 32 = -72 + 32 = -40°F

°F = 125°C * 9/5 + 32 =  1125/5 + 32 = 225 + 32 = 257°F

−40 < 5 over 9 (F − 32) < 125; −40 < F < 257

Answer; −40 < 5 over 9 (F − 32) < 125; −40 < F < 257

The volume Equals Four thirds pi r cubed of a spherical balloon changes with the radius. a. At what rate ​(cubed​/in​) does the volume change with respect to the radius when r equals 9 in question mark b. Using the rate from part a​, by approximately how much does the volume increase when the radius changes from 9 to 9.5 in question mark

Answers

Answer:

Step-by-step explanation:

Given

Volume of spherical Balloon is given by

V=(4)/(3)\pi r^3

(a)Rate of change of balloon w.r.t to radius is given by

\frac{\mathrm{d} V}{\mathrm{d} r}=3* (4)/(3)\pi r^2=4\pi r^2

at r=9\ in.

\frac{\mathrm{d} V}{\mathrm{d} r}=324\pi\ in.^2

(b)Using the rate Volume change when radius increases from r=9 in. to r=9.5 in.

\Delta V=4\pi (9)^2(9.5-9)=162\pi\ in.^3

Plz help me I need so much help

Answers

Answer:

  see below

Step-by-step explanation:

We have to assume that the question is referring to the doubly-shaded length, the one that is 10 bright blue units long.

You can read the overall length of the shaded area as 5/4, from the markings.

For the next part, you need to count the number of units that are shaded bright blue (10) and the number that are shaded blue overall (15). As a fraction of those shaded blue overall, the ones shaded bright blue are 10/15 = 2/3.

So, the length shaded bright blue is 2/3 of the overall length of 5/4. It is represented by the product ...

  \boxed{(2)/(3)*(5)/(4)}